Difference between revisions of "1950 AHSME Problems/Problem 49"

(Solution)
m (Problem)
Line 1: Line 1:
 
==Problem==
 
==Problem==
  
A triangle has a fixed base <math>AB</math> that is <math>2</math> inches long. The median from <math>A</math> to side <math>BC</math> is <math> 1\frac{1}{2}</math> inches long and can have any position emanating from <math>A</math>. The locus of the vertex <math>C</math> of the triangle is:
+
A triangle has a fixed base <math>AB</math> that is <math>2</math> inches long. The median from <math>A</math> to side <math>BC</math> is <math> 1\frac{1}{3}</math> inches long and can have any position emanating from <math>A</math>. The locus of the vertex <math>C</math> of the triangle is:
  
 
<math>\textbf{(A)}\ \text{A straight line }AB,1\dfrac{1}{2}\text{ inches from }A \qquad\\
 
<math>\textbf{(A)}\ \text{A straight line }AB,1\dfrac{1}{2}\text{ inches from }A \qquad\\

Revision as of 17:34, 3 June 2020

Problem

A triangle has a fixed base $AB$ that is $2$ inches long. The median from $A$ to side $BC$ is $1\frac{1}{3}$ inches long and can have any position emanating from $A$. The locus of the vertex $C$ of the triangle is:

$\textbf{(A)}\ \text{A straight line }AB,1\dfrac{1}{2}\text{ inches from }A \qquad\\ \textbf{(B)}\ \text{A circle with }A\text{ as center and radius }2\text{ inches} \qquad\\ \textbf{(C)}\  \text{A circle with }A\text{ as center and radius }3\text{ inches} \qquad\\ \textbf{(D)}\ \text{A circle with radius }3\text{ inches and center }4\text{ inches from }B\text{ along } BA \qquad\\ \textbf{(E)}\ \text{An ellipse with }A\text{ as focus}$

Solution

The locus of the median's endpoint on $BC$ is the circle about $A$ and of radius $1\frac{1}{2}$ inches. The locus of the vertex $C$ is then the circle twice as big and twice as far from $B$, i.e. of radius $3$ inches and with center $4$ inches from $B$ along $BA$ which means that our answer is: $\textbf{(D)}$.


Solution 2

Let $A(a,y_A)$, $B(b,y_B)$ and $C(x,y)$.

Hence, $D\left(\frac{x+b}{2},\frac{y+y_B}{2}\right)$ is a midpoint of $BC$.

Thus, the equation of needed locus is \[\left(\frac{x+b}{2}-a\right)^2+\left(\frac{y+y_B}{2}-y_A\right)^2=\left(\frac{3}{2}\right)^2,\] which is equation of the circle: \[(x-(2a-b))^2+(y-(2y_A-y_B))^2=3^2.\]

Thus, D) is valid because \[\sqrt{(2a-b-b)^2+(2y_A-y_B-y_B)^2}=2\sqrt{(a-b)^2+(y_A-y_b)^2}=2AB=4.\]

See Also

1950 AHSC (ProblemsAnswer KeyResources)
Preceded by
Problem 48
Followed by
Problem 50
1 2 3 4 5 6 7 8 9 10 11 12 13 14 15 16 17 18 19 20 21 22 23 24 25 26 27 28 29 30 31 32 33 34 35 36 37 38 39 40 41 42 43 44 45 46 47 48 49 50
All AHSME Problems and Solutions

The problems on this page are copyrighted by the Mathematical Association of America's American Mathematics Competitions. AMC logo.png